2002 AMC 10B Problems/Problem 7

Revision as of 14:02, 27 December 2008 by Temperal (talk | contribs) (solution, and box)

Problem

Let $n$ be a positive integer such that $\frac {1}{2} + \frac {1}{3} + \frac {1}{7} + \frac {1}{n}$ is an integer. Which of the following statements is not true?

$\mathrm{(A) \ } 2\text{ divides }n\qquad \mathrm{(B) \ } 3\text{ divides }n\qquad \mathrm{(C) \ } 6\text{ divides }n\qquad \mathrm{(D) \ } 7\text{ divides }n\qquad \mathrm{(E) \ } n>84$

Solution

Writing the first four fractions with a common denominator, we have $\frac{41}{42}+\frac{1}{n}$, hence $n=42$ is a solution. Thus, our answer is $\boxed{(E)}$.

See Also

2002 AMC 10B (ProblemsAnswer KeyResources)
Preceded by
Problem 6
Followed by
Problem 8
1 2 3 4 5 6 7 8 9 10 11 12 13 14 15 16 17 18 19 20 21 22 23 24 25
All AMC 10 Problems and Solutions